From: W. Trevor King Date: Thu, 12 Apr 2012 02:00:39 +0000 (-0400) Subject: Fix begin/end problem/solution errors in some S&J v4 problems. X-Git-Url: http://git.tremily.us/?a=commitdiff_plain;h=f95a68a5dafee9acb295179f9f60c2038c112a53;p=course.git Fix begin/end problem/solution errors in some S&J v4 problems. --- diff --git a/latex/problems/Serway_and_Jewett_4/problem05.34.tex b/latex/problems/Serway_and_Jewett_4/problem05.34.tex index b3e5a00..0d95faa 100644 --- a/latex/problems/Serway_and_Jewett_4/problem05.34.tex +++ b/latex/problems/Serway_and_Jewett_4/problem05.34.tex @@ -21,4 +21,4 @@ So = \ans{-3.596\E{6}\U{N}} \end{equation} where the $-$ sign indicates an attractive force. -\begin{solution} +\end{solution} diff --git a/latex/problems/Serway_and_Jewett_4/problem05.50.tex b/latex/problems/Serway_and_Jewett_4/problem05.50.tex index b5678da..e4949ac 100644 --- a/latex/problems/Serway_and_Jewett_4/problem05.50.tex +++ b/latex/problems/Serway_and_Jewett_4/problem05.50.tex @@ -10,6 +10,7 @@ tabletop revolves. What are \Part{c} the speed of the puck? \end{problem*} % problem 5.50 +\begin{solution} \Part{a} Constructing a free body diagram for $m_2$, we see that the only forces on it are the tension \vect{T} and gravity $\vect{F}_{g2}$. diff --git a/latex/problems/Serway_and_Jewett_4/problem19.40.tex b/latex/problems/Serway_and_Jewett_4/problem19.40.tex index ca047a3..a3766d3 100644 --- a/latex/problems/Serway_and_Jewett_4/problem19.40.tex +++ b/latex/problems/Serway_and_Jewett_4/problem19.40.tex @@ -1,4 +1,4 @@ -\begin{problem}{19.40} +\begin{problem*}{19.40} An insulating solid sphere of radius $a$ has a uniform volume charge density $\rho$ and carries a total positive charge $Q$. A spherical gaussian surface of radius $r$, which shares a common center with the diff --git a/latex/problems/Serway_and_Jewett_4/problem21.45.tex b/latex/problems/Serway_and_Jewett_4/problem21.45.tex index 449baf8..228425d 100644 --- a/latex/problems/Serway_and_Jewett_4/problem21.45.tex +++ b/latex/problems/Serway_and_Jewett_4/problem21.45.tex @@ -109,4 +109,4 @@ So using $V_c' = V_c/10$ we have \ln(10) &= \frac{t}{RC} \\ t &= RC\ln(10) = \ans{8.29\U{$\mu$s}} \end{align} -\ens{solution} +\end{solution}